LSAT and Law School Admissions Forum

Get expert LSAT preparation and law school admissions advice from PowerScore Test Preparation.

 AnnBar
  • Posts: 33
  • Joined: Mar 24, 2017
|
#39085
Hello PowerScore,

When i attempted this question I was between A C and D. I diagrammed it [promise to yield insight into causes of practical problems that affects people's quality of life :arrow: obligation to promote research]

Answer A: seemed to be the contrapositive [NO obligation to promote research :arrow: NO insight into...]

Answer C: seemed to fit the original diagram [promise to yield insight into causes of practical problems that affects people's quality of life :arrow: obligation to promote research]

Answer D: seemed to also fit the original diagram.

Then I went back and tried to look more closely at the "practical problems that affect people's quality of life" and see which of the contenders best fit that.

A- didn't fit "causes of practical problems"
C- the research position also doesn't seem to fit "causes of practical problems"
E- CHOSE IT bc it talks about structures that cause disease which fit more "causes of practical problems"

I just wanted to double check if my thought process was correct or if there was a faster/easier way I could have gotten to the answer.

Thank you,
AB
 AthenaDalton
PowerScore Staff
  • PowerScore Staff
  • Posts: 296
  • Joined: May 02, 2017
|
#39420
Hi Ann,

Your thought process looks fine -- it led you to the right answer, after all!

One way to narrow down the options more quickly is to focus on projects that directly match the principle (as opposed to the principle's contrapositive). We know for certain that research which will solve practical problems should be funded, but we're only guessing as to the criteria for which programs should not be funded. The university president doesn't say that research should be funded only if it solves practical problems. There could be some research worth funding that falls short of that -- we just don't know!

I hope this helps speed things along for you -- good luck studying!

Athena Dalton
 jeremiah230!!
  • Posts: 9
  • Joined: Nov 27, 2019
|
#76518
I am having trouble understanding the explanations given for this question here. Can you tell me if this is also a good way of solving this question?

Given the conditional principle in the stimulus, there are only 2 possible conclusions that can be derived from it: we can either trigger the sufficient condition and then justify funding a research project; OR, we can negate the necessary – by saying that there is NO obligation to promote research - and thus conclude that said research does not satisfy our sufficient condition. Thus, an answer choice that contains a situation in which an institution denies funding is incorrect; while not contradicting the stimulus, such an answer would not JUSTIFY that it is okay to deny funding from the conditional in the stimulus, which is about an OBLIGATION to fund.

However, because the necessary condition is an obligation to fund, an answer choice which depicts a situation in which funding is granted will be correct so long as the sufficient conditions are met as well; if they are then the institution has an obligation to provide funding, and is thus justified in doing so.

Is this a good way of seeing this problem?

EDIT:: I hadn't seen the post above this, but seeing that has given me a bit more clarity. How I see it, we could justify funding research if it satisfies the sufficient condition of the conditional in the stimulus; however, using that conditional we cannot justify DENYING funding for research. At best, we can say that it doesn't GO AGAINST that principle.
User avatar
 cornflakes
  • Posts: 48
  • Joined: Feb 19, 2021
|
#85315
Hello,

I have read through the explanations in the thread a few times and am still struggling to see how A could not pass as the contrapositive - it seems as though a few other students felt the same.

The answer reads: "A university denies a grant application from a faculty member for work on a solution to a famous mathematical puzzle that has no relation to practical concerns."

Original Diagram: If insights into practical problems --> (obligation to) promote research.

In answer A, we clearly have a negation of both elements - the denied grant application functions as the action of not promoting the research (and hence not having the obligation to) and the description at the end spells that it does not offer insights into practical problems. The issue I'm having in in the way this answer is worded is there is not a clear indicator in how to interpret the sufficient and necessary conditions. For example, if we inserted a "because" before the "no relation to practical concerns" segment, this would then demonstrate clearly that it should be interpreted as - no practical concerns --> no promotion of research - which would be a mistaken negation. Alternatively, if we inserted a "so it follows that" in the same spot, then it would be completely reasonable to assume its saying - no promotion of research --> no practical concerns. Instead, it is reading to me just as a fact set of the two negations - which leaves the door open for either mistaken negation or valid contrapositive.

Appreciate any insight here.
User avatar
 Ryan Twomey
PowerScore Staff
  • PowerScore Staff
  • Posts: 141
  • Joined: Mar 04, 2021
|
#85699
Hey Cornflakes,

With principle must be true questions where the stimulus is conditional in nature, we must assign the parts of the answer choices as necessary and sufficient. The way I do this is I say that the outcome is the necessary condition in the answer choices.

With answer choice A: I would say the outcome is denying a grant, which would fit into our conditional statement as not promoting. The outcome would be the necessary condition. There was no practical concern thus they did not promote. So this does not fit our conditional statement in the stimulus because it is in the incorrect order.

It can be hard to determine which part of the answer choice is the outcome in this type of must be true principle question, but I would highly recommend practicing at least 10-15 in a row to get the feel for how to assign sufficient and necessary in the answer choices of a principle question.

Now with answer choice E, this fits our conditional statement. There were practical problems at play, and thus they promoted the study. Fitting our conditional statement of: causes practical problems ------>promote

Basically the largest takeaway is, you need to assign the parts of the answer choices as sufficient and necessary in principle must be true questions if there is a conditional statement in the stimulus above. The way I do this is I assign what seems to be the outcome of the answer choice as the necessary condition.

I hope this explanation helps you.

Best,
Ryan

Get the most out of your LSAT Prep Plus subscription.

Analyze and track your performance with our Testing and Analytics Package.